Δυνατή άσκηση

Συντονιστές: AΝΔΡΕΑΣ ΒΑΡΒΕΡΑΚΗΣ, silouan, rek2

Άβαταρ μέλους
Lymperis Karras
Δημοσιεύσεις: 170
Εγγραφή: Παρ Νοέμ 06, 2020 5:16 pm

Δυνατή άσκηση

#1

Μη αναγνωσμένη δημοσίευση από Lymperis Karras » Πέμ Απρ 01, 2021 3:07 pm

Έστω ABC ισοσκελές τρίγωνο, με CA=CB, M το μέσον της AC και H το ίχνος του ύψους από την κορυφή C στην AB. Ο κύκλος που περνά από τα B,C,M τέμνει την CH στο σημείο Q. Αν CQ=m, να εκφραστεί η ακτίνα του περιγεγραμμένου κύκλου του \triangle ABC συναρτήσει του
m.


Ένας μαθηματικός χρειάζεται μολύβι, γόμα και μεγάλο καλάθι αχρήστων.
-Hilbert

Λέξεις Κλειδιά:
Άβαταρ μέλους
george visvikis
Επιμελητής
Δημοσιεύσεις: 13273
Εγγραφή: Παρ Νοέμ 01, 2013 9:35 am

Re: Δυνατή άσκηση

#2

Μη αναγνωσμένη δημοσίευση από george visvikis » Πέμ Απρ 01, 2021 5:38 pm

Lymperis Karras έγραψε:
Πέμ Απρ 01, 2021 3:07 pm
Έστω ABC ισοσκελές τρίγωνο, με CA=CB, M το μέσον της AC και H το ίχνος του ύψους από την κορυφή C στην AB. Ο κύκλος που περνά από τα B,C,M τέμνει την CH στο σημείο Q. Αν CQ=m, να εκφραστεί η ακτίνα του περιγεγραμμένου κύκλου του \triangle ABC συναρτήσει του
m.
Ωραία άσκηση!


Έστω N το μέσο του BC. Το περίκεντρο O του ABC είναι το σημείο τομής της CH με τη μεσοκάθετο του BC.

Επειδή η HC είναι μεσοκάθετος των AB, MN θα είναι QA=QB και QM=QN. Αλλά, η CQ διχοτομεί

τη γωνία M\widehat CB, οπότε QB=QM, δηλαδή το Q είναι το περίκεντρο του ABNM.
Δυνατή άσκηση.png
Δυνατή άσκηση.png (24.48 KiB) Προβλήθηκε 982 φορές
Αν λοιπόν B' είναι το αντιδιαμετρικό του B ως προς αυτό τον κύκλο, τότε επειδή O\widehat NB=90^\circ, τα σημεία N,

O, B' θα είναι συνευθειακά. Άρα O είναι το βαρύκεντρο του CBB', οπότε \displaystyle CO = \frac{2}{3}CQ \Leftrightarrow \boxed{ R = \frac{{2m}}{3}}


Μιχάλης Τσουρακάκης
Δημοσιεύσεις: 2769
Εγγραφή: Παρ Ιαν 11, 2013 4:17 am
Τοποθεσία: Ηράκλειο Κρήτης

Re: Δυνατή άσκηση

#3

Μη αναγνωσμένη δημοσίευση από Μιχάλης Τσουρακάκης » Πέμ Απρ 01, 2021 6:49 pm

Lymperis Karras έγραψε:
Πέμ Απρ 01, 2021 3:07 pm
Έστω ABC ισοσκελές τρίγωνο, με CA=CB, M το μέσον της AC και H το ίχνος του ύψους από την κορυφή C στην AB. Ο κύκλος που περνά από τα B,C,M τέμνει την CH στο σημείο Q. Αν CQ=m, να εκφραστεί η ακτίνα του περιγεγραμμένου κύκλου του \triangle ABC συναρτήσει του
m.
Λόγω της μεσοκαθέτου AH είναι MQ=QB=QA=r

Από Πτολεμαίο στο CMQB έχουμε br+ \dfrac{b}{2}r=mMB \Rightarrow MB= \dfrac{3br}{2m}

(ABC)=2(MAB)  \Rightarrow \dfrac{ab^2}{4R}  =2  \dfrac{ \dfrac{3b^2ra}{2m} }{4r} \Rightarrow R= \dfrac{2m}{3}
ακτίνα.png
ακτίνα.png (21.61 KiB) Προβλήθηκε 959 φορές
τελευταία επεξεργασία από Μιχάλης Τσουρακάκης σε Πέμ Απρ 01, 2021 9:38 pm, έχει επεξεργασθεί 1 φορά συνολικά.


Άβαταρ μέλους
Lymperis Karras
Δημοσιεύσεις: 170
Εγγραφή: Παρ Νοέμ 06, 2020 5:16 pm

Re: Δυνατή άσκηση

#4

Μη αναγνωσμένη δημοσίευση από Lymperis Karras » Πέμ Απρ 01, 2021 7:41 pm

Πραγματικά, και οι δύο λύσεις με εντυπωσίασαν. ΣΥΓΧΑΡΗΤΗΡΙΑ, διότι αυτή η άσκηση ήταν δύσκολη, αλλά ταυτόχρονα και πολύ όμορφη, με πολλές διαφορετικές λύσεις! Θα επιχειρήσω κάτι διαφορετικό, αν και το πήγαινα προς την λύση του κ, Βισβίκη.


Ένας μαθηματικός χρειάζεται μολύβι, γόμα και μεγάλο καλάθι αχρήστων.
-Hilbert
Άβαταρ μέλους
silouan
Επιμελητής
Δημοσιεύσεις: 1398
Εγγραφή: Τρί Ιαν 27, 2009 10:52 pm

Re: Δυνατή άσκηση

#5

Μη αναγνωσμένη δημοσίευση από silouan » Πέμ Απρ 01, 2021 7:51 pm

Μπορείτε να δείτε εδώ για άλλες λύσεις. Είναι το δεύτερο θέμα της JBMO 2004.
https://artofproblemsolving.com/communi ... 344p101576


Σιλουανός Μπραζιτίκος
Απάντηση

Επιστροφή σε “Γεωμετρία - Προχωρημένο Επίπεδο (Juniors)”

Μέλη σε σύνδεση

Μέλη σε αυτήν τη Δ. Συζήτηση: Δεν υπάρχουν εγγεγραμμένα μέλη και 2 επισκέπτες